Đến nội dung

Ego nội dung

Có 288 mục bởi Ego (Tìm giới hạn từ 02-06-2020)



Sắp theo                Sắp xếp  

#627238 Tìm các số nguyên dương $n$ để đa thức sau không thể phân tích thàn...

Đã gửi bởi Ego on 15-04-2016 - 13:11 trong Đa thức

Tìm các số nguyên dương $n$ để đa thức sau không thể phân tích thành các đa thức hệ số nguyên:

a) $x^{n} + 4y^{n}$
b) $x^{n} + 4y^{n + 2}$

Irreducible 1




#621476 Chứng minh rằng tồn tại $1000$ số nguyên dương liên tiếp sao cho tr...

Đã gửi bởi Ego on 20-03-2016 - 20:47 trong Số học

Vừa đọc được một bài toán trên AoPS, nhớ đến bài toán này, ứng dụng của định lý giá trị trung gian.
Chứng minh rằng tồn tại $1000$ số nguyên dương liên tiếp sao cho trong chúng chỉ có đúng $69$ số nguyên tố.

Spoiler




#620831 Tìm các số nguyên dương x sao cho $2x^2+4x+1$ là số chính phương.

Đã gửi bởi Ego on 17-03-2016 - 21:55 trong Số học

Viết lại ta có $2(x + 1)^{2} - 1 = y^{2} \iff y^{2} - 2(x + 1)^{2} = -1$ là pt Pell loại 2.



#627235 ... q | $a^p - 1$ và q | $a^n - 1$

Đã gửi bởi Ego on 15-04-2016 - 12:55 trong Số học

Bài toán không đúng với $a = 2$, mình nghĩ $(a, p, n) = (2, 3, 5)$ là một ví dụ. Nếu đề bài sửa lại thành $a \ge 3$ thì ta chỉ việc chọn $q$ là ước nguyên tố của $a - 1$. Từ đó có $q\mid a - 1 \mid a^{p} - 1$ và $q\mid a - 1 \mid a^{n} - 1$.




#626586 Tìm GTNN và GTLN của các biểu thức

Đã gửi bởi Ego on 11-04-2016 - 18:08 trong Bất đẳng thức - Cực trị

Cho $a_{1}, a_{2}, \cdots , a_{n}$ là một bộ hoán vị của $n$ số nguyên dương đầu tiên. Tìm GTNN và GTLN của các biểu thức:
a) $P = \frac{a_{1}}{a_{2}} + \frac{a_{2}}{a_{3}} + \cdots + \frac{a_{n - 1}}{a_{n}}$
b) $Q = \frac{a_{1}}{a_{2}} + \frac{a_{2}}{a_{3}} + \cdots + \frac{a_{n - 1}}{a_{n}} + \frac{a_{n}}{a_{1}}$




#626593 Tìm GTNN và GTLN của các biểu thức

Đã gửi bởi Ego on 11-04-2016 - 18:56 trong Bất đẳng thức - Cực trị

Xin phép được xử câu a) trước :D
Xét hai bộ $(1 < 2 < \cdots < n)$ và $(\frac{1}{n} < \frac{1}{n - 1} < \cdots < \frac{1}{1})$
Theo bất đẳng thức sắp thứ tự, $n = 1\times \frac{1}{1} + 2\times \frac{1}{2} + \cdots + n\times \frac{1}{n} \le \sum_{i = 1}^{n - 1}a_{i}\times \frac{1}{a_{i + 1}} \le 1\times \frac{1}{n} + 2\times \frac{1}{n - 1} + \cdots + n\times \frac{1}{1} = n + \frac{n - 1}{2} + \frac{n - 2}{3} + \cdots + \frac{1}{n}$
 




#624576 $$\sum \frac{a^{2}b}{c} +...

Đã gửi bởi Ego on 03-04-2016 - 19:59 trong Bất đẳng thức và cực trị

Cho $a \ge b \ge c \ge d > 0$. Chứng minh rằng
$$\frac{a^{2}b}{c} + \frac{b^{2}c}{d} + \frac{c^{2}d}{a} + \frac{d^{2}a}{b} \ge a^{2} + b^{2} + c^{2} + d^{2}$$




#621205 Giải phương trình nghiệm nguyên $x^{3}-y^{2}=2$

Đã gửi bởi Ego on 19-03-2016 - 19:13 trong Số học

Bổ đề : Cho $a,b$ nguyên tố cùng nhau và thỏa $(a^2+b^2) \vdots p$ khi đó $4|(p-1)$ ($a,b$ là các số nguyên và $p$ là một số nguyên tố)
Áp dụng : Phương trình tương đương với $x^3-1=1+y^2$ . Dễ thấy $gcd(y,1)=1$ suy ra $x^3-1$ có ước nguyên tố dạng $4k+1$ suy ra nó cũng có dạng $4m+1$ (nếu có dạng $4m+3$ thì phải có ước nguyên tố dạng $4l+3$ vô lí) (với $k,m,l$ là các số nguyên)
Suy ra $x^3-1 \equiv 1 \pmod{4}$
Suy ra $x^3 \equiv 2 \pmod{4}$ (vô lí)
Vậy phương trình vô nghiệm.

Không đúng đâu em. Bổ đề em chỉ đúng cho số nguyên tố lẻ thôi. Nếu có sự tham gia của $p = 2$ thì VT có dạng $4k + 2$ và không có mâu thuẫn đâu nhé



#614080 Đa thức

Đã gửi bởi Ego on 10-02-2016 - 23:23 trong Đa thức

Cho đa thức $P(x)$ bậc $n$ hệ số thực thỏa mãn $P(x)$ nhận giá trị nguyên tại ít nhất $n + 1$ điểm nguyên liên tiếp.
a) Chứng minh rằng tất cả các hệ số của $P(x)$ là hữu tỷ.
b) Có nhất thiết phải nguyên không?
c) Chứng minh rằng với mọi $x$ nguyên thì $P(x)$ nguyên.

p.s: Bài này do mình chế, cách đặt câu hỏi chưa hay lắm. Chủ yếu là câu c), mong các bạn cho ý kiến  :luoi:




#623012 Chứng minh phương trình $\sum_{i=0}^{n}a_{...

Đã gửi bởi Ego on 27-03-2016 - 20:01 trong Số học

Đề bài không thiếu dữ kiện $a_{0}$ mà thiếu dữ kiện $n$. Ví dụ $x + 1$ có nghiệm hữu tỷ nhé. Đề bài đúng là $n \ge 2$. Và dữ kiện $a_{0}$ thật sự không cần thiết vì nếu $a_{0} = 0$ ta suy ra ngay luôn $\overline{a_{n}a_{n - 1}\cdots 0} \vdots 10$ không là số nguyên tố.
Mình giải 1 lần trên AoPS rồi. Mình có tổng quát nhưng lười tìm lại link quá, sẽ giải TH cơ số 10 ở đây.
Giả sử ngược lại là $P(x) = a_{n}x^{n} + \cdots + a_{0}$ có nghiệm hữu tỷ. Nghĩa là $P(x) = (ax - b).Q(x)$ với $a \in \mathbb{Z}^{+}, b \in \mathbb{Z}$, $\text{gcd}(a, b) = 1$.
Để ý là theo bổ đề Bezout ta sẽ có $a\mid a_{n}$ và $b\mid a_{0}$ nên $0 \le a, b \le 9$
Thực hiện phép chia đa thức ta sẽ thu được $Q(x) \in \mathbb{Z}[x]$
Nhận xét: nếu $b$ không âm thì $P(x)$ sẽ có nghiệm hữu tỷ không âm, điều này là vô lí do các hệ số của ta đều không âm. Do đó $b < 0$. Thay $-b = c$ với $c > 0$ cho thuận tiện, ta có $P(x) = (ax + c).Q(x)$ với $a, c \in \mathbb{Z}^{+}$
Ta có $p = P(10) = (10a + c).Q(10)$. Do $10a + c > 1$ và $p \in \mathbb{P}$ nên $10a + c = p$ và $Q(10) = 1$
Do $0 \le a, c \le 9$ nên $10a + c$ là biểu diễn của $p$ trong cơ số $10$ (đây là biểu diễn duy nhất) nên từ đó ta suy ra $P(x)$ bậc $1$. Vô lí.




#617568 Giải PTNN: $a^n+b^n=p^x$

Đã gửi bởi Ego on 28-02-2016 - 23:06 trong Số học

Ý bạn là tìm $n$ luôn?
Đặt $p^{u} = \text{gcd}(a, b)$. Ta có $a'^{n} + b'^{n} = p^{x - un}$ với $\text{gcd}(a', b') = 1$. Dễ thấy $x - un \ge 1$.
i) $n = 1$. Khi đó ta có bộ nghiệm $(a, b, x, p, n) = (a, p^{x} - a, x, 1)$.
ii) $n \ge 2$
Nếu $a = b = 1$, ta cũng có một bộ nghiệm
Nếu $(a, b, x, p, n) = (2, 1, 2, 3, 3)$
Ngoại trừ các trường hợp trên, $a^{n} + b^{n}$ có ước nguyên tố $q$ sao cho $q\nmid a + b$, điều này sẽ dẫn đến $a + b = 1$, vô lí.




#621367 Hỏi có bao nhiêu số thực $x$ thỏa mãn $|f(x)| = 1$?

Đã gửi bởi Ego on 20-03-2016 - 10:55 trong Đa thức

Cho đa thức $f(x) = (x - 1)(x - 2)^{2}(x - 3)^{3}\cdots (x - 2000)^{2000}$. Hỏi có bao nhiêu số thực $x$ thỏa mãn $|f(x)| = 1$?




#620765 Chứng minh b=d

Đã gửi bởi Ego on 17-03-2016 - 18:07 trong Số học

ii) Có $a + b + c - d\mid b(a + b + c - d) - b^{2} + 2bd - d^{2} \implies a + b + c - d \mid (b - d)^{2}$
Vì $a + b + c - d$ là ước nguyên tố nên $a + b + c - d\mid |b - d|$
Nếu $b > d$ thì $a + c + b - d \mid b - d$ với $0 < b - d < a + c + b - d$, vô lí.
Nếu $b < d$ thì $a + b + c - d \mid d - b$. Đặt $d - b = u(a + b + c - d) \implies u(a + c) = (u + 1)(d - b) \implies \frac{a + c}{d - b} = 1 + \frac{1}{u} \le 2$ (do $d > b$ nên $u \ge 1$). Do đó $a + c \le 2(d - b) \iff a + 2b + c \le 2d \implies (a + 2b + c)^{3} \le 8d^{3}$
Mặt khác, $(a + 2b + c)^{3} \ge 54abc = 54d^{3}$ theo AM-GM. Từ đó suy ra $54d^3 \le 8d^{3}$ suy ra $d = 0$, vô lí.
Vậy $b = d$



#625242 Biểu diễn dưới dạng $x^2+y^2-z^2$ với $x,y,z$ là $3...

Đã gửi bởi Ego on 05-04-2016 - 22:12 trong Số học

Thật ra điều kiện $x < y$ không quan trọng. Ta sẽ thay đổi vị trí $x, y$ nếu cần. Chỉ cần quan tâm $x \neq y$ và $\max(x, y) < z$
i) Nếu $n = 1$ chọn $1 = 4^{2} + 7^{2} - 8^{2}$
ii) Nếu $n = 2$ (cái này tìm mãi không có nghiệm :'()
iii) Nếu $n = 3$ chọn $3 = 4^{2} + 6^{2} - 7^{2}$
iv) Nếu $n = 7$ (cái này cũng vậy, nếu điều kiện sửa lại $x \ge y < z$ thì tìm được)
v) Nếu $n = 2k + 3 \; (k \neq 0, 2)$
Chọn $\begin{cases}x = 3k + 2 \\ y = 4k \\ z = 5k + 1\end{cases} \implies x^{2} + y^{2} - z^{2} = 2k + 3$
Khi đó $x \neq y$ và $3k + 2 < 4k < 5k + 1$
vi) Nếu $n = 2k + 2$ thì chọn $\begin{cases} x = 3k - 1\\ y = 4k + 1 \\ z = 5k\end{cases} \implies x^{2} + y^{2} - z^{2} = 2k + 2$




#622984 về các số $7^{2n}+5.7^n+1$

Đã gửi bởi Ego on 27-03-2016 - 17:51 trong Số học

Không ai giải hết  :( 
Lời giải của mình. Gọi $p$ là một ước nguyên tố của $7^{4k + 2} + 5.7^{2k + 1} + 1$. Nhận xét $p\neq 7$ và $p\neq 3$ và $p$ lẻ:
i) Ta có $p\mid (7^{2k + 1} + 1)^{2} + 21.7^{2k} \implies p\mid [7^{-k}(7^{2k + 1} + 1)]^{2} + 21 \implies \left(\frac{-21}{p}\right) = 1$
ii) Lại có $p\mid 4(7^{2n} + 5.7^{n} + 1) = (2.7^{n} + 5)^{2} - 21 \implies \left(\frac{21}{p}\right) = 1$
Kết hợp lại, ta có $1 = \left(\frac{-21}{p}\right) = \left(\frac{21}{p}\right).\left(\frac{-1}{p}\right) = \left(\frac{-1}{p}\right)$.
Từ đó ta suy ra $p \equiv 1\pmod{4}$
Chứng minh hoàn tất. $\bigstar$




#630999 Cho $p$ là số nguyên tố lớn hơn $2$.CMR: $\left...

Đã gửi bởi Ego on 03-05-2016 - 18:03 trong Số học

Dạo này topic Số học THCS toàn bài ghê ghê nhỉ ._.
Đặt $a = \sqrt{5} + 2$ và $b = \sqrt{5} - 2$. Theo F.L.T cho $\gcd(p, 2) = 1$, ta có $2^{p + 1} \equiv 4\pmod{p}$
Ta cần chứng minh $\left\lfloor a^{p}\right\rfloor \equiv 4\pmod{p}$
Nhận xét là $0 < b^{p} < 1$
Do đó $a^{p} - b^{p} < a^{p} < a^{p} - b^{p} + 1$. Mặt khác, dễ dàng chứng minh $a^{p} - b^{p}$ là số nguyên.
Từ đó ta có $\left\lfloor a^{p}\right\rfloor = a^{p} - b^{p} = \left(\sqrt{5} + 2\right)^{p} - \left(\sqrt{5} - 2\right)^{p}$
Xét trên modulo $p$, dùng bổ đề cơ bản $(m + n)^{p} \equiv m^{p} + n^{p}\pmod{p}$
Áp dụng, ta có $a^{p} - b^{p} \equiv (\sqrt{5})^{p} + 2^{p} - (\sqrt{5})^{p} + 2^{p} \equiv 2^{p + 1} \equiv 4\pmod{p}$.
Xong. $\bigstar$




#633867 Chứng minh rằng $AO \perp MN$

Đã gửi bởi Ego on 18-05-2016 - 12:16 trong Hình học

Cho tam giác $ABC$ nhọn có $H$ là trực tâm. $D, E, F$ là chân đường cao hạ từ $A, B, C$, theo thứ tự. $BE$ cắt $DF$ tại $N$ và $BF$ cắt $DE$ tại $M$. Gọi $O$ là tâm của $\odot (BHC)$. Chứng minh rằng $AO \perp MN$




#610640 $a^2-b!=2416$

Đã gửi bởi Ego on 23-01-2016 - 22:45 trong Số học

$a^{2} = b! + 2416$
Nếu $b \ge 11$ thì VP có thể đồng dư $7$ modulo $11$
Và vế trái có thể đồng dư $0; 1; 4; 9; 5; 3$. Vô lí
Vậy $b < 11$. Bước tiếp theo là thử và sai.



#630310 $\prod_{a = 1}^{k}\prod_{b = 1}^...

Đã gửi bởi Ego on 30-04-2016 - 09:09 trong Số học

Chứng minh rằng với mỗi số tự nhiên $k$ thì $$\prod_{a = 1}^{k}\prod_{b = 1}^{k}\prod_{c = 1}^{k}\frac{a + b + c - 1}{a + b + c - 2}$$ là số nguyên.

Nguồn: AoPS - trumpeter




#617247 Chứng minh rằng $B$ có thể chọn số $k$ khéo léo để chỉ mộ...

Đã gửi bởi Ego on 27-02-2016 - 20:17 trong Số học

Hôm trước mình làm sai, mình làm lại như sau:

Giả sử các phần tử của $S$ là $a_1,a_2,...a_n$ và tất cả các ước nguyên tố của các $a_i$ là $p_1,p_2,...p_k$

và $a_1=p_1^{m_{11}}p_2^{m_{12}}...p_k^{m_{1k}}$, ... $a_i=p_1^{m_{i1}}p_2^{m_{i2}}...p_k^{m_{ik}}$

Ta sẽ chọn $K=p_1^{n_1}p_2^{n_2}...p_k^{n_k}$ với các $n_i$ thíc hợp

Khi đó $d(Ka_i)=(n_1+m_{i1}+1)(n_2+m_{i2}+1)...(n_k+m_{ik}+1)$

kiểu bài này là dùng $CRT$ luôn , gọi ra tập $P$ gồm $n.k$ số nguyên tố $q_{i1},q_{i2},...q_{ik}$ với $i$ chạy từ $1$ đến $n$

Với mỗi $j$ từ $1$ đến $k$ ta chọn $n_j$ sao cho $n_j+m_{ij}+1$ chia hết cho $q_{ij}$ , $i$ chạy từ $1$ đến $n$, và ko chia hết cho các số nguyên tố còn lại.

Vì chỉ có hữu hạn các tổng $n_j+m_{ij}+1$ nên ta có thể chọn các $q_{ij}$ đủ lớn để ko có $2$ số nào cùng chia hết cho $1$ số nguyên tố thuộc $P$

Khi đó mỗi tích  $q_{i1}q_{i2}...q_{ik}$ chỉ chia hết duy nhất $d(Ka_i)$. Vì thế $B$ chỉ cần một lượt chơi để tìm ra số $a_i$ .

 $đpcm$

Mình cũng làm vậy :-P




#615562 Chứng minh rằng $B$ có thể chọn số $k$ khéo léo để chỉ mộ...

Đã gửi bởi Ego on 17-02-2016 - 18:45 trong Số học

Cho một tập hữu hạn $S$ gồm các số nguyên dương. Cả hai người A và B đều biết trước các phần tử của $S$. Họ thực hiện trò chơi như sau: A lấy một số $m$ bất kì thuộc $S$ (không để B biết); tiếp theo B chọn một số $k$ bất kì (không nhất thiết thuộc $S$); sau đó A tính toán và cho $B$ biết giá trị của $d(km)$ với $d(n)$ là số các ước số dương của $n$ để B đoán ra số $m$; làm như vậy cho đến khi B đoán đúng. Chứng minh rằng $B$ có thể chọn số $k$ khéo léo để chỉ một lượt chơi là B có thể tìm được số như yêu cầu.

p.s. Đây là đề qgia của Ukraina năm nay. Mình có một lời giải nhưng không nghĩ nó hay cho lắm, đăng lên để anh em tham khảo cho ý kiến.



#614116 Tặng VMF

Đã gửi bởi Ego on 11-02-2016 - 09:51 trong Đa thức

Bài này mình làm thử vài trường hợp nhỏ rồi tổng quát lên, đưa lên AoPS hỏi cũng chưa có trả lời :-) Hôm nay có ý, đưa mọi người xem thử.
a) Cho tam thức bậc hai $P(x) = ax^{2} + bx + c$ thỏa mãn $|P(x)| \le 1 \; \forall x \in [-1; 1]$. CMR $P'(x) \le 4 \; \forall x \in [-1; 1]$
b) Cho đa thức bậc ba $P(x) = ax^{3} + bx^{2} + c + d$ thỏa mãn $|P(x)| \le 1 \; \forall x \in [-1; 1]$. CMR $P'(x) \le 9 \; \forall x \in [-1; 1]$.
c) Cho đa thức bậc $n$: $P(x) = a_{n}x^{n} + \ldots + a_{0}$ thỏa mãn $|P(x)| \le 1 \; \forall x \in [-1; 1]$. CMR $P'(x) \le n^{2} \; \forall x \in [-1; 1]$




#612475 CMR: Không tồn tại số nguyên $a$ sao cho $f(a) = 2024$

Đã gửi bởi Ego on 02-02-2016 - 19:05 trong Đa thức

Không thấy ai giải hết :| Phần đa thức Olympic thấy trống vắng nhỉ :-?. Mình xử bài này vậy :-P. Đặt
$$f(x) = 2013 + (x - a)(x - b)(x - c)(x - d).Q(x)$$
với $a, b, c, d$ là các số nguyên phân biệt và $Q(x) \in \mathbb{Z}[x]$
Giả sử ngược lại là tồn tại $L$ để $f(L) = 2024$. Khi đó $11 = (L - a)(L - b)(L - c)(L - d).Q(L)$
TH1. $Q(L) = 11$ (TH $Q(L) = -11$ lí luận tương tự): Khi đó có 4 số $mnpq = 1$, khi đó có 2 số sẽ bằng nhau, vô lí.
TH2. $Q(L) = 1$ (TH $Q(L) = -1$ lí luận tương tự): Khi đó có 4 số $mnpq = 11$, cũng lí luận được sẽ có 2 số bằng nhau, vô lí. :-)




#624790 Tìm số thực $a$ lớn nhất sao cho với mọi số tự nhiên $n>1...

Đã gửi bởi Ego on 04-04-2016 - 18:01 trong Số học

Nói cách khác, ta sẽ tìm giá trị nhỏ nhất của $f(n) = \frac{\sigma (n)}{\sqrt{n}.\tau (n)}$ với $n \in \mathbb{Z}^{+}$
Đặt $n = p_{1}^{a_{1}}.p_{2}^{a_{2}}\cdots p_{k}^{a_{k}}$, khi đó ta viết lại $f(n)$ như sau:
$$f(n) = \frac{(1 + p_{1} + \cdots + p_{1}^{a_{1}})(1 + p_{2} + \cdots + p_{2}^{a_{2}})\cdots (1 + p_{k} + \cdots + p_{k}^{a_{k}})}{(1 + a_{1})(1 + a_{2})\cdots (1 + a_{k})\sqrt{p_{1}^{a_{1}}p_{2}^{a_{2}}\cdots p_{k}^{a_{k}}}}$$
Bổ đề. Nếu $p \ge 3$ là số nguyên tố thì $\frac{1 + p + \cdots + p^{a}}{(1 + a)\sqrt{p^{a}}} \ge 1$. (*)



Áp dụng bổ đề ta có $f(p_{1}^{a_{1}}p_{2}^{a_{2}}\cdots p_{k}^{a_{k}}) \ge f(p_{1}^{a_{1}}p_{2}^{a_{2}} \cdots p_{k - 1}^{a_{k - 1}}) \ge \cdots \ge f(p_{1}^{a_{1}})$

  1. Nếu $p_{1} \ge 5$ thì áp dụng bổ đề $f(p_{1}^{a_{1}}) = g(a_{1}) \ge g(1) = \frac{1 + p}{2\sqrt{p}} \ge \frac{6}{2\sqrt{5}}$.
  2. Nếu $p_{1} = 2$ thì áp dụng bổ đề nếu $a_{1} \ge 3$ thì $f(2^{a_{1}}) \ge  f(2^{3})$ với $a_{1} \ge 3$. Với $a_{1} = 1$ thì $f(2^{1}) = \frac{3}{2\sqrt{2}}$. Với $a_{1} = 2$ thì $f(2^{2}) = \frac{7}{6}$.
    So sánh ta thấy $f(2) \le f(n) \; \forall n \ge 2$. Dấu bằng xảy ra khi và chỉ khi $n = 2$.

Vậy $a = f(2)$.
 

 

 




#631015 Tích ba số tự nhiên liên tiếp là lũy thừa của một số nguyên với số mũ không n...

Đã gửi bởi Ego on 03-05-2016 - 19:38 trong Số học

Hỏi có tồn tại tích ba số tự nhiên liên tiếp là lũy thừa của một số nguyên với số mũ không nhỏ hơn $2$?